Sistemas de ecuaciones e inecuaciones no lineales

2000020307

Parte: 
A
Halla el conjunto de todos los pares ordenados de números reales de la forma \([x;y]\) que son soluciones de la siguiente ecuación. \[ \frac{x-7}{y+1}=5 \] ¿Cuál de las siguientes opciones es correcta?
\[ \left\{ \left[5m+12;m\right];m\in\mathbb{R}\setminus \left\{-1\right\}\right\} \]
\[ \left\{ \left[x;0.2x-2.4\right];x\in\mathbb{R}\setminus \left\{-0.7\right\}\right\} \]
\[ \left\{ \left[5a-12;a\right];a\in\mathbb{R}\setminus \left\{-1\right\}\right\} \]
\[ \left\{ \left[q;0.2q+2.4\right];q\in\mathbb{R}\setminus \left\{-1.8\right\}\right\} \]

2000020305

Parte: 
A
Halla el conjunto de todos los pares ordenados de números reales de la forma \(\left[x;y\right] \) que son soluciones de la siguiente ecuación. \[\frac{y+2}{x-4}=3\] ¿Cuál de las siguientes opciones es incorrecta?
\[ \left\{ \left[2b;b+\frac{14}{3}\right];b\in\mathbb{R}\setminus \left\{2\right\}\right\} \]
\[ \left\{ \left[x;3x-14\right];x\in\mathbb{R}\setminus \left\{4\right\}\right\} \]
\[ \left\{ \left[\frac{y+14}{3};y\right];y\in\mathbb{R}\setminus \left\{-2\right\}\right\} \]
\[ \left\{ \left[\frac{a}{3};a-14\right];a\in\mathbb{R}\setminus \left\{12\right\}\right\} \]

2000020304

Parte: 
B
Resuelve el siguiente sistema de ecuaciones en el conjunto de los números reales. \[\begin{aligned} x-y&=2\\ x^2-y^2&=2\\ \end{aligned}\] Elige la opción correcta.
El sistema tiene exactamente una solución.
El sistema no tiene solución.
El sistema tiene infinitas soluciones.
El cociente entre los números \(x\) e \(y\) es \(3\).

2000020303

Parte: 
A
Resuelve el siguiente sistema de ecuaciones en el conjunto de los números reales. \[\begin{aligned} x+y&=4+\frac{1}{27}\\ x\cdot y&=\frac{4}{27}\\ \end{aligned}\] Elige la opción correcta.
\(|x-y|=\frac{107}{27}\)
El sistema tiene exactamente una solución.
El sistema no tiene solución.
El sistema tiene infinitas soluciones.

2000020302

Parte: 
A
Resuelve el siguiente sistema de ecuaciones en el conjunto de los números reales. \[\begin{aligned} x^2+y&=2\\ 2x-y+3&=0\\ \end{aligned} \] Elige la opción correcta.
Los números \(x\) e \(y\) son opuestos entre sí.
La suma de los números \(x\) e \(y\) es igual a \(-2\).
La media aritmética de los números \(x\) e \(y\) es igual a \(2\).
La razón entre los números \(x\) e \(y\) es \(2:1\).

2000020301

Parte: 
C
Resuelve el siguiente sistema de ecuaciones en el conjunto de los números reales. \[ \begin{aligned} x+y&=-5\\ 1+\sqrt{2x+4y}&=\sqrt{x+3y}\\ \end{aligned}\] Elige la opción correcta.
\(x=-12,\ y=7\)
\(x=12,\ y=7\)
El sistema no tiene solución.
El sistema tiene infinitas soluciones.

2010011206

Parte: 
C
Dado el sistema \[\begin{aligned} y & = \frac{k} {x}, & & \\y & = a, & & \end{aligned}\] donde \(a\), \(k\) son parámetros reales y \(x\), \(y\) son variables reales. Halla las condiciones para que el sistema tenga la única solución en \(\mathbb{R}^{+}\times \mathbb{R}^{-}\).
\(a < 0\) y \(k < 0\)
\(a < 0\) y \(k > 0\)
\(a > 0\) y \(k < 0\)
\(a > 0\) y \(k > 0\)

2010006704

Parte: 
B
Suponiendo que \(c\in \mathbb{R}\) halla la condición para que el siguiente sistema tenga dos soluciones en \(\mathbb{R}\times \mathbb{R}\). \[ \begin{alignedat}{80} &x^{2} & + &2y^{2} & = 6 & & & & & & \\ &x & + &y & = c & & & & & & \\\end{alignedat}\]
\(|c| < 3\)
\(|c| =3\)
\(|c| > 3\)
\(|c| \in \mathbb{R}\)